Đến nội dung

apollo_1994 nội dung

Có 91 mục bởi apollo_1994 (Tìm giới hạn từ 25-05-2020)



Sắp theo                Sắp xếp  

#228502 Zải zùm

Đã gửi bởi apollo_1994 on 09-02-2010 - 11:50 trong Bất đẳng thức và cực trị

Cho 3 số dương nhỏ hơn a;b;c <1 . CMR:
a(1-c)+b(1-a)+c(1-b)<1

$(1-a)(1-b)(1-c)+abc>0 \Rightarrow đpcm $



#191848 xin các bạn và thày cô giải giúp!

Đã gửi bởi apollo_1994 on 03-10-2008 - 14:56 trong Hình học

1/ cho Tam giác ABC có góc B = 70 độ, góc C =50 độ. Đường cao AH, trung tuyến AM.
Tính góc HAM.

2/ Cho tam giác ABC cân tại A, góc A=20 độ, I là trung điểm của AC.
Tính góc IBC.

Mấy bài toán tính góc này mình học hồi lớp 7.Nói chung mấy bài này thì thường là người ta lấy 1 cạnh đã cho làm cạnh rồi dựng các tam giác vuông cân,tam giác đều,tam giác nửa đều... ,nối linh tinh lại,sau đó tìm các tam giác bằng nhau có liên quan đến góc cân tính....
Có thể tham khảo thêm trong sách "Nâng cao và phát triển toán 7" của Vũ Hữu Bình,trong đó có hẳn 1 chuyên đề về vấn đề này(nếu như mình nhớ không nhầm^^)



#216518 Tổ Hợp mới nè !

Đã gửi bởi apollo_1994 on 07-10-2009 - 19:34 trong Tổ hợp - Xác suất và thống kê - Số phức

Trên mặt mẳng cho 2009 đường thẳng cắt nhau tạo thành m tam giác nhọn. Tìm giá trị lớn nhất của m.

Mò mẫm.....Không biết đúng hay sai a_{n}
NX: 3 đường thẳng bất kỳ chỉ tạo với nhau tối đa là 1 tam giác.
Ta chứng minh n đường thẳng bất kỳ tạo với nhau không quá $\dfrac{(n-2)(n-1)n}{6} :D$ tam giác
Thật vậy:với $n=1;2;3;4$ đúng
Giả sử nó đúng với k đường thẳng $d_1,d_2,..,d_k$
Xét 1 đuờng thẳng $d_{k+1}$.Đường thẳng này sẽ cắt tối đa là k đường thẳng còn lại và khi đó số tam giác mới tạo thêm bởi $(d_{n+1};d_i,d_j)$ sẽ là $\dfrac{n(n-1)}{2}$ (điều này dễ thấy vì có $k$ cách chọn $d_i$ và $k-1$ cách chọn $d_j$)
Vậy số tam giác tối đa tạo bởi $k+1$ đường thằng là:
$\dfrac{(k-2)(k-1)k}{6} +\dfrac{k(k-1)}{2}= \dfrac{(k-1)k(k+1)}{6}$ đúng
Áp dụng a_{n} với $n=2009$-->done



#221015 tìm n

Đã gửi bởi apollo_1994 on 20-11-2009 - 20:13 trong Số học

tìm n để $ 3^{1024}-1 \vdots2^n$

Ta chứng minh $a_m=3^^2^m-1$ chia hết cho $2^{m+2}$ nhưng không chia hết cho $2^{m+3}$
Thật vậy,giả sử đúng với $m=k$
$a_{k+1}=(3^^2^k-1)(3^^2^k+1)=a_k.(3^^2^k+1) \vdots (2^{k+2}.2)=2^{k+3}$ nhưng không chia hết cho $2^{k+4}$ do $a_k \not \vdots 2^{n+3}$ và $3^^2^k+1 \not \vdots 4$
Áp dụng với $m=10$ thì n nhận giá trị tư 1 đến 12.
Không hiểu diễn đàn mình gõ lũy thừa tầng cái kiểu gì mà số mũ bị gió thổi bay vù vù thế kia :D



#255424 Trời 8-3 mà không có ai!

Đã gửi bởi apollo_1994 on 20-03-2011 - 19:22 trong Phương trình, hệ phương trình và bất phương trình

Ngắn hơn nè !
$ \Leftrightarrow 2{\left( {4x + 2} \right)^2} = \sqrt {2x + 15} + 28$
Đặt $\sqrt {2x + 15} = 4y + 2$
$\begin{array}{l} \Leftrightarrow 16{y^2} + 16y + 4 = 2x + 15\\ \Leftrightarrow 16{y^2} + 16y = 2x + 11\left( 2 \right)\end{array}$
Từ PT đầu $32{x^2} + 32x = 4y + 22\left( 3 \right)$
$ \Rightarrow \left( 2 \right)\left( 3 \right)$ có hệ :
$\left\{ \begin{array}{l}32{x^2} + 32x = 4y + 22\\16{y^2} + 16y = 2x + 11\end{array} \right.$
Đây là hệ đối xứng nè !

Câu hỏi : Những phương trìhh như thế nào thì làm được như thế này ? :P



#198984 toán cực trị!

Đã gửi bởi apollo_1994 on 27-05-2009 - 09:57 trong Bất đẳng thức và cực trị

Nhờ các bạn giải giùm với!!!
Tìm Max P= (a-b)4+ (b-c)4+ (c-a)4
Cho a,b,c là các số thực không nhỏ hơn 1 và không lớn hơn 2.

Từ giả thiết suy ra $0 \leq |a-b|,|b-c|,|c-a| \leq 1$
Do đó $P \leq |a-b|+|b-c|+|c-a|$
Không mất tổng quát giả sử $a \geq b,a \geq c$
Khi đó $P \leq 2a-b-c-|b-c|$
+Nếu $b \geq c$ thì $P \leq 2(a-b) \leq 2(2-1)=1$
+Nếu $b \leq c$ tương tự
Vậy $maxP=2$ khi có 2 số bằng 2,1 số bằng 1



#214776 Thảo luận các bài toán Bất đẳng thức trên tạp chí THTT

Đã gửi bởi apollo_1994 on 20-09-2009 - 20:03 trong Toán học & Tuổi trẻ

Trước tiên sẽ là các bài toán của tháng 5 /2009 và 6/2009 (đã hết hạn gửi bài =)) )

Bài 3 (T4/384) (Trịnh Xuân Tình)Cho $x,y,z$ là các số không âm và thỏa mãn $x+y+z=1$.Tìm giá trị lớn nhất của biểu thức
$P=(x+2y+3z)(6x+3y+2z)$

Hơ vẫn còn bài 3 này :)
$P/2=(x+2y+3z)(3x+ \dfrac{3}{2}y+z) \leq \dfrac{[4(x+z)+ \dfrac{7}{2}y]^2 }{4}=\dfrac{(4-\dfrac{y}{2})^2 }{4}\leq 4^2/4=4 $
do $y \geq 0$
$\Rightarrow P \leq 8 $



#206109 thách mọi người đấy

Đã gửi bởi apollo_1994 on 22-07-2009 - 21:44 trong Số học

Cho x,y,z nguyên dương, nguyên ttố cùng nhau và 1/x+1/y=1/z
hỏi x+y có phải là số chính phương hok?

$z(x+y)=xy \Rightarrow xy \vdots z \Rightarrow z=1 \Rightarrow xy=x+y$
$\Rightarrow \not \exists x,y$
$\Rightarrow x+y$ không CP :D
Vô lý chỗ nào không nhỉ :D
p/s: title kêu như chuông :D



#206184 thách mọi người đấy

Đã gửi bởi apollo_1994 on 23-07-2009 - 16:51 trong Số học

Sorry :geq, đọc đề cứ nghĩ (x;y)=(y;z)=(z;x)=1 :D
Hóa ra là (x;y;z)=1 à :P



#191813 Số vô tỉ

Đã gửi bởi apollo_1994 on 02-10-2008 - 14:27 trong Số học

Chứng minh rằng
a=0,123456789......( sau dấu phẩy là các số tự nhiên liên tiếp ) là một số vô tỉ

Bài này có ở đâu đó trên diễn đàn rùi
Sơ sơ là thế này:Giả sử số đó có dạng 0,a(b) với chu kì b gồm k chữ số
Xét số $10^{2k}$ có trong phần thập phân ấy=> phần thập phân gồm toàn các chữ số 0->vô lý



#203925 PTvo tỉ

Đã gửi bởi apollo_1994 on 04-07-2009 - 19:51 trong Phương trình, hệ phương trình và bất phương trình

Cho d,e,f nguyên thỏa mãn: $d+e\sqrt[3]{2}+f\sqrt[3]{4}=0$
CMR: d=e=f=0

Chỉ cần d,e,f hữu tỉ là OK!
Làm thế này: :)
$d+e\sqrt[3]{2}+f\sqrt[3]{4}=0 \Rightarrow (d+e\sqrt[3]{2}+f\sqrt[3]{4})(f\sqrt[3]{2}-e)=0$
$\Rightarrow (df-e^2)\sqrt[3]{2}=de-2f^2$
$\Rightarrow df-e^2=de-2f^2=0$
....



#216513 PTNN

Đã gửi bởi apollo_1994 on 07-10-2009 - 19:00 trong Số học

Tìm nghiệm nguyên dưong của pt: $x^2-y^3=7$ Mình post bên phần phưong trình mà kô thấy ai trả lời hết các bạn giải hộ mình.

$x^2+1=y^3+8=(y+2)(y^2-2y+4)$
Vì $x^2+1$ chia $4$ dư $1$ hoặc $2$ nên y bắt buộc phải chia $4$ dư $1$(bạn tự c/m).
Suy ra $y+2$ chia $4$ dư $3$,do đó $y^3+8$ luôn có 1 ước nguyên tố dạng $4k+3$.
$\Rightarrow x^2+1 \vdots 4k+3\Rightarrow 1 \vdots 4k+3$
vô lý
PT vô nghiệm.



#194477 pt

Đã gửi bởi apollo_1994 on 06-12-2008 - 20:57 trong Phương trình, hệ phương trình và bất phương trình

<spam>Nếu bạn muốn lời giải gọn nhất thì hãy xem trong TTT tháng sau.Còn bài này chưa hết hạn gửi,bạn không nên post lên đây như vậy.



#198580 Phần nguyên

Đã gửi bởi apollo_1994 on 24-05-2009 - 21:25 trong Kinh nghiệm học toán

Mấy anh ơi, em lớp 9, mún bik nhiều về phần nguyên và phương pháp, em nghe nói TTT có chuyên đề jì về phần nguyên hay lắm nhưng em ko đặt báo, anh nào có kinh nghiệm truyền thụ cho em nha :))Em sắp thi òi :D
có ebooks càng tốt ạ :D

Bạn tìm trong quyển số của bộ sách ĐHSP ý,có hẳn 1 chuyên đề về cái này.Trong TTT cũng chỉ có 1 số VD xoay kĩ về 1 dạng bài trong đó thui.
Nói thật là vô cùng đau sốt với cái thể loại phần nguyên này :Leftrightarrow



#200532 Phương trình nghiệm nguyên, số chính phương

Đã gửi bởi apollo_1994 on 07-06-2009 - 08:10 trong Số học

giải PT nghiệm nguyên dương: $2(y+z)=xyz-x$

Với $x=1$ thì $2(y+z)=yz-1$ giải bình thường :)
Với $x \geq 2$ thì $2(y+z) \geq 2yz-2 \Rightarrow (y-1)(z-1) \leq 2$
mà nó lại nguyên dương...



#200318 Phương trình nghiệm nguyên, số chính phương

Đã gửi bởi apollo_1994 on 05-06-2009 - 21:53 trong Số học

cho n là số tự nhiên .Tìm n sao cho :$n^{2}+n+2 \vdots 2005$

$n^{2}+n+2 \not \vdots 5 \forall n \in N$ nên $\not \exists n$
Còn bài bên trên là đề thi KHTN năm nào đấy :),xét số dư của 1 số CP cho 17 là OK!



#200145 Phương trình nghiệm nguyên, số chính phương

Đã gửi bởi apollo_1994 on 04-06-2009 - 16:36 trong Số học

giải PT nghiệm nguyên : $x^{2}+y^{2}+z^{2}=x^{2}y^{2}$

Ông anh này tự sướng à :D
Bài trên thì cóa thể làm như vầy:
+Nếu $x^2y^2$ lẻ thì $z^2$ lẻ
=> VT chia 4 dư 3,Vp chia 4 dư 1=>vô no
+Nếu $x^2y^2$ chẵn ,giả sử x chẵn thì $y^2+z^2 \vdots 4 \Rightarrow y \vdots 2,z \vdots 2$
Tới đây xuống thang,pt có nghiệm duy nhất $(0;0;0)$



#200860 Phương trinh và hệ phương trình

Đã gửi bởi apollo_1994 on 09-06-2009 - 19:28 trong Phương trình, hệ phương trình và bất phương trình

Giải PT: $\sqrt{x^{2}+12}+5=3x+\sqrt{x^{2}+5}$

Bài này thì cơ bản roài :
$\Leftrightarrow \sqrt{x^{2}+12}-4=(3x-6)+(\sqrt{x^{2}+5}-3)$
$\Leftrightarrow \dfrac{x^2-4}{\sqrt{x^{2}+5}+3} +3(x-2)=\dfrac{x^2-4}{\sqrt{{x^{2}+12}}+4}$
$\Leftrightarrow x=2$ (dễ thấy $x>0$)



#200540 Phương trinh và hệ phương trình

Đã gửi bởi apollo_1994 on 07-06-2009 - 09:46 trong Phương trình, hệ phương trình và bất phương trình

1, Giải PT: $\sqrt[5]{x-1} + \sqrt[3]{x+8}=x^{3}+1 $

Không có ai giải thì đề nghị chú tự sướng bài 1 cái :)



#234519 Olympic khu vực duyên hải Bắc Bộ 2010

Đã gửi bởi apollo_1994 on 15-04-2010 - 19:09 trong Thi HSG cấp Tỉnh, Thành phố. Olympic 30-4. Đề thi và kiểm tra đội tuyển các cấp.

Hình như Chuyên Hùng Vương không tham gia...



#191849 Nhìn thì dễ nhưng......

Đã gửi bởi apollo_1994 on 03-10-2008 - 15:17 trong Phương trình, hệ phương trình và bất phương trình

$\dfrac{1}{5 \sqrt[3]{3}-2 \sqrt[3]{9}+1 }= \dfrac{\sqrt[3]{3}}{(5\sqrt[3]{3}+6)(\sqrt[3]{3}-1)}$
Nhân liên hợp là ok



#218504 new new new

Đã gửi bởi apollo_1994 on 25-10-2009 - 17:28 trong Bất đẳng thức - Cực trị

Hic,nhìn em lại nhớ đến bài này(câu 5 vòng 1 thi vào chuyên hùng vương)

Cho $x,y,z>0 $thỏa mãn $xyz=x+y+z+2$
Chứng minh rằng:
$\dfrac{1}{\sqrt{xy}}+\dfrac{1}{\sqrt{yz}}+\dfrac{1}{\sqrt{zx}}\leq\dfrac{3}{2}$
Bài này thì đặt kiểu gì nhỉ?



#357833 Một số bài về ma trận và định thức

Đã gửi bởi apollo_1994 on 30-09-2012 - 16:22 trong Đại số tuyến tính, Hình học giải tích

Bài 1: Cho ma trận
$A=\begin{bmatrix} 1 & x &..& x^{n-1} & x^n\\ x & x^2 &..& x^{n} & 1 \\ ...\\ x^n & 1& ...& x^{n-2} & x^{n-1} \end{bmatrix}$
Tính $det(A)$

Bài 2: Cho ma trận
$A=\begin{bmatrix}
1&2&3&m\\ 1&m&2&3 \\ 1&2&m&3\\m&1&2&3 \end{bmatrix}$
Biện luận $r(A)$ theo m.

Bài 3: Tính giá trị lớn nhất của định thức cấp 5 chỉ nhận các phần tử $1$ và $-1$.

Bài 4:
Tính $\begin{bmatrix}a&1&0\\ 0&a&1 \\ 0&0&a\end{bmatrix}^{100}$



#262677 Một số bài tập liên quan đến định lý giá trị trung gian, định lý Lagrange, Roll

Đã gửi bởi apollo_1994 on 29-05-2011 - 22:40 trong Phương trình hàm

Bài 1: Giải phương trình:
$7^x+5^x=8^x+4^x$

Bài 2: Cho hàm số $f(x),g(x)$ liên tục trên $[a,b]$ và số $\alpha$ thỏa mãn
$\dfrac{f(x)}{g(x)}>\alpha \forall x\in[a,b]$
CMR: $\dfrac{f(x)}{g(x)}\geq \alpha+1 \forall x\in[a,b] $

ps: bài 2 có thể mình ko nhớ chính xác đề



#228286 Một Phương pháp Phân tích bình phương SOS

Đã gửi bởi apollo_1994 on 05-02-2010 - 22:12 trong Bất đẳng thức và cực trị

Nhưng tại sao dùng nó để phân tích bài này thì lại không được?
$(a^2+ab+b^2)(b^2+bc+c^2)(c^2+ca+a^2)-3(a^2b^2+b^2c^2+c^2a^2)(ab+bc+ca)$
Cho $a=b$ : $3a^2c^2(a-c)^2$
..Tính được $S_a= \dfrac{3}{2}b^2c^2$...
Cuối cùng bấm máy-->sai
Ai giải thích giùm.
Hay mình tính sai nhỉ?